1. Trang chủ
  2. » Khoa Học Tự Nhiên

Tuyển tập đề thi vô địch bất đẳng thức thế giới P3

30 465 4
Tài liệu được quét OCR, nội dung có thể không chính xác
Tài liệu đã được kiểm tra trùng lặp

Đang tải... (xem toàn văn)

Tài liệu hạn chế xem trước, để xem đầy đủ mời bạn chọn Tải xuống

THÔNG TIN TÀI LIỆU

Thông tin cơ bản

Định dạng
Số trang 30
Dung lượng 1,05 MB

Nội dung

Trang 1

First solution:

Using the identity (a + 6)(6+ c)(c +a) = (a+6+4+c)(ab+ be + ca) — 1 we reduce

the problem to the following one ab + be + ca + — > — > 4 atb+e Now, we can apply the AM-GM Inequality in the following form ,/ (ab + be + ca)? b+6 an ————— ——— 9(a+b+)

And so it is enough to prove that

(ab + be + ca)? > 9(at+b+c)

But this is easy, because we clearly have ab + be + ca > 3 and (ab+ be + ca)? > 3abc(a + b + e) = 3(a+b+ ©) Second solution: 3 We will use the fact that (a + b)(b+ c)(e +a) > gia + b+e)(ab + be + ca) 2 So, it is enough to prove that 9 (0b + be + ca) + > 1 Using the AM-GM ; a+b+c Inequality, we can write „| (œb + be + ca)? ————— >1 a+b+c7— 8l(a+b+ec) — ? 2 g (ab + be + ca) + because

(ab + be + ca)2 > 3abe(a + b + e) = 3(a+ b+ e)

57 Prove that for any a,b,c > 0,

(a7 + b` + c?)(a+b— e)(b+e— a)(e+a— b) < abe(ab + be + ca)

Solution:

Trang 2

58 | D.P.Mavlo | Let a,b,c > 0 Prove that 1 161 1)(b + 1)(c + 1 Btatbtesn4 rere Sal fy gle We Mer) b b Mes Dery) 1 + abe Kvant, 1988 Solution: The inequality is equivalent to the following one La Dato — _>939=————“^= eatery — abe + 1 or 1 2 a

abe Ya+ — +) a c+ 5 > 2(Soa+ am) But this follows from the inequalities 2 b 2 C 2 a œ“bc + P > 2ab, “ca + — > 2bc,c“ab + ọ > 2ca a and 5 1 2 1 9 1 a-c+ — > 2a,b°a+ — > 2b,c°b+ = > 2c e a b

Trang 3

Of course, this is true for any other variable, so we can add all these inequalities to obtain that

which is the desired inequality

60 Let a,b,c,d > 0 such that ø + b+ e= 1 Prove that 1 1 a® +6° + + abed > min { 5, stat: Kvant, 1993 Solution: 1 Suppose the inequality is false Then we have, taking into account that abc < 2m 1 1 1 the inequality d (5 — abe) >@+P43 - 3 We may assume that abe < 7 Now, 1 we will reach a contradiction proving that a® + 6? + c? + abed > T It is sufficient to prove that 1 a+h+e—— 1 ———T—————— aửe +a3+b”+c7> + 7 abc But this inequality is equivalent to + l5abc > 1 We use now the identity 1+ 9ab » a3 = 3abe+1—3 ` ab and reduce the problem to proving that » ab< _ which is Schur’s Inequality

61 Prove that for any real numbers a, b,c we have the inequality

3 +22)? +b2)?(ø— e)?(b— ø)” > (1+a”)(1+ð2)(1+c”)(a— b)”(b— e)”(e— a)°

AMM

Solution: 5 52

(1 1

The inequality can be also written as » ren? assume that a,b,c are distinct) Now, adding the inequalities

(1+ a?)(1 + 07) (+6)d+e) J, 14+ 06?

(1+c?)(a—b)2 — (1+a@?)(6-c)? ~ |a— b|le— Ù|

(which can be found using the AM-GM Inequality) we deduce that (1+ a7)(1+ 0?) 14+ 6?

2>axzza- >> I6-s0-a

Trang 4

and so it is enough to prove that the last quantity is at least 1 But it follows from » 1+? jy SS 1 + - |l(b— a)(b—e)|— —c) and the problem is solved = 1

Trang 5

— — = (e+w+z~3) > “—(3ÿwz— 3) =0 Equality holds for # = = z = 1 Remark:

Using the substitution 6 = a+1(6> 2) andz= -, y= Đ z= - (abc = 1) the

inequality becomes as follows

1 1 1 3

a? (b +c) 7 b2(e + a) 7 đ*{(a + b) 2 2

For 6 = 3, we obtain one of the problems of IMO 1995 (proposed by Russia) 63 Prove that for any real numbers 21, ,2%n,Y1, -,Yn Such that 2?+ -+22 = yee dyad, k=1 ?ì (Z1a — #s1)” < 3 ( — > een Korea, 2001 Solution: We clearly have the inequality +? hr n 2

(amram YS wan? = (Set) (Sout)- (Sam) = 1<i<j<n i=1 i=1 i=1

- (1- Soa) mm) i=1 i=1

Because we also have mới <1, we find immediately that i=1 ( — 3 su) ụ + year] <2 ( — ` ea] i=1 =1 z—=1 and the problem 1s solved

64 [ Laurentiu Panaitopol | Let ai, ø›, , „ be pairwise distinct positive inte- gers Prove that

2n+1

aj tag to +a, 2S (ai + as + - + an)

Trang 6

Solution:

Without loss of generality, we may assume that a, < ag < - < a, and hence a; > 17 for all 7 Thus, we may take b; = a; —i > 0 and the inequality becomes nr nr n 9 < ; mín + 1)(2n + 1) _ 2n +1 > : ; —————————, n(n + 1)(2n 4+ 1) ` bj +25 ib; + 6 >—a › Ù¡ + 6 i=1 i=1 +1

Now, using the fact that aj, > a; we infer that 6) < ¿ < - < ð„ and from Chebyshev’s Inequality we deduce that w=1 +1 7=1 and the conclusion is immediate Also, from the above relations we can see imme- diately that we have equality if and only if a,,a2, ,a@, is a permutation of the numbers 1,2, ,n 65 [ Calin Popa ] Let a,b,c be positive real numbers such that a+b+c¢= 1 Prove that b/c + ca + avb ` 3v3 a(V3c+vab) b(V3a+vbe) c(V3b+ ca) — 4 - Solution: Rewrite the inequality in the form bc > 3v3 ca + 4 ——+a b ¬ be ca ab With the substitution « = 4/—,y = „—'®=Ứ= the conditlon a + b-+e= 1 a c becomes xy + yz + zx = 1 and the inequality turns into > 3v “4 # 2 V3y + yz But, by applying the Cauchy-Schwarz Inequality we obtain 2 a » 7 ) 3 ty 3v3 » V3ay + ayz > V34 3xryz > V3 + i =—, 4 v3

where we used the inequalities

Trang 7

66 | Titu Andreescu, Gabriel Dospinescu | Let a, 6, c,d be real numbers such that

(1 + a?)(1 + 67)(1 +c?)(1 + d?) = 16 Prove that

—3 < ab+bce+cd+da+ac+ bd — abcd < 5

Solution:

Let us write the condition in the form 16 = H¿ +a)- Ta —i) Using symmetric

sums, we can write this as follows

16= (1-10 a- Sab +i > abe + abcd) (1+% 0 a- Sabi D abe + abcd)

So, we have the identity 16 = (1 — Ð` ab + abcd)? + (SS a— >> abc)” This means that |1 — S> ab+ abcd| < 4 and from here the conclusion follows

67 Prove that

(a? + 2)(b + 2)(c? + 2) > 9(ab + be + ca)

for any positive real numbers a, 6, c

APMO, 2004 First solution:

We will prove even more: (a? + 2)(Ù2 + 2)(c? +2) > 3(œ + b + c)? Because (x+b+e) < (la| + |b| + |c|)”, we may assume that a,b,c are nonnegative We will use the fact that if x and y have the same sign then (1+ 2)(1+y) >1+2+y So,

we write the inequality in the form

(+2) > othe

and we have three cases 2

2_ 4 2_ 4 ( a)

i) If a,b,c are at least 1, then TT (“ 3 +1) >14505 5 » 2)

Trang 8

Second solution:

Expanding everything, we reduce the problem to proving that

(abc)? + 23 `a?0? +45 0a? +8> 9S — ab

Because 3300? > 3 `ab and 23 `a?0? +6> 4À ` ab, we are left with the inequality (abc)? + » a?+2> 2À ` ab Of course, we can assume that a,b,c are non-negative and we can write a = 27,b = y’,c = z” In this case

2S ab-S oa? =(z++z)(z+— z)(u+z— z)(z+z— 9)

It is clear that if x,y,z are not side lengths of a triangle, then the inequality is trivial Otherwise, we can take x =u+v,y =v+w,z = w-+u and reduce the inequality to

((u+v)(v+w)(w+u))*+2> 16( + 0 + u)uU

We have ((u+v)(v + w)(w +u))* +141>3%/(ut vs (vtw)t(utw)! and

it remains to prove that the last quantity is at least 16(u+v+w)uvw This comes

down to 3

16

(utv)i(v+w)*(w+u)* > sp (wap) (u ++u)

But this follows from the known inequalities 8 (utv)(vt+w)(w+u) > gute +w)(uv + vw + wu), 8 (uv + vw + wu)* > 34(uow)3, wu + +~+ > 3u) Third solution: In the same manner as in the Second solution, we reduce the problem to proving that (abe)? +2> 23 `ab— » Now, using Schur’s Inequality, we infer that 9ab 23 2ab—À `a?< Thưa

Trang 9

68 | Vasile Cirtoaje | Prove that IÍ 0 < z < < z and z+ + z = zz +2, then a) (1— zø)(1— z)(1— zz) 3 0; b) ay < 1a’y? < = Jays lay" < 55 Solution: a) We have (l—ay)(1—yz) =1l-—ay—yz+ay?2z =1—a2y—yz+y(@t+y+2—-2) =(y-1* >0 and similarly (1—øz)—zz) =(1—z)“>0, (1—zz)(1— zu) = (L— ø) >0

So the expressions 1 — xy, 1 — z and 1 — zz have the same sign

b) We rewrite the relation x+y+z = xyz+2 as (1—x)(1-y)+(1-z)(1—-zy) = 0 Ifz > 1 then z > # > z > 1 and so (I — z)(1 — ø) + (1— z)(1— zg) > 0, impossible So we have x < 1 Next we distinguish two cases 1) xy < 1; 2) zy > I1

32 1) ty <1 We have ay <a <1and ay’ <a<1< 5

2) xy > 1 From y > ,/ry we get y > 1 Next we rewrite the relation x + y+2z=

0z + 2 as # + T— 2 = (z0 — 1)z Because z > # gives # + — 2 > (z0 — l)U, (@T— 1)(2—#— z) > 0 so 2> z(1+ 0) Using the AM-GM Inequality, we have

1+ > 2Vÿ and 1+ = 1+ 548 5301-32 Thus we have 2 > 2e,/y and 2 32 2>3z = which means that xy < 1 and xy? < 7 32 The equality x?y = 1 takes place when x = y = 1 and the equality x?y”? = 2z 2

takes place when x = 3° y= r=2

Trang 10

which is the same as 12(y+z) > 5+9y?+82?+ 1l2yz @ (2z—-1)?4+ (8y+2z—-2)? < 0,

which is clearly impossible Thus, the conclusion follows

70 | Gabriel Dospinescu, Marian Tetiva ] Let x,y,z > 0 such that

z + +Z—+xUz Prove that

(— 1)(w— 1)(z— 1) < 63 - 10

First solution:

Because of x < xyz > yz > 1 (and the similar relations xz > 1, zy > 1) at most

one of the three numbers can be less than 1 In any of these cases (2 < 1, y > 1, z > 1 or the similar ones) the inequality to prove is clear The only case we still have to analyse is that when x > 1, y >1 and z> 1

In this situation denote x-l=a,y-—1=6,z-l=c Then a,b,c are nonnegative real numbers and, because zœ=a+l,=b+l,z=c+], they satisfy a+1+b6+1+c+1=(a4+1)(b4+1) (c+), which means abe + ab+act+ be = 2 Now let « = Wabc; we have ab + ac + be > 3V abache = 32’, that’s why we get a + 34? <2 (x41) (x? 4+ 22-2) < 06

& (x +1) (ec +1+ V3) (ec +1- V3) <0

Trang 11

Second solution:

Like in the first solution (and due to the symmetry) we may suppose that x > 1, y > 1; we can even assume that « > 1, y > 1 (for x = 1 the inequality is plain) Then we get xy > 1 and from the given condition we have +%+1 — 1` The relation to prove is (z — 1)(T— 1) — 1) < 6v3-— 10 âđ â 2#z (s + œz + yz) < 6V3 — 9,

or, with this expression of z,

day" — ay zy—-1 — (ex +y) —** <6V3-9 6 xy —1 & (xy —x —y)* + (6V3 — 10) zụ < 6/3 -— 9,

after some calculations

Now, we put x =a+1,y=0+41 and transform this into a2b? + (6v3 10) (a +b-+ab) —2ab>0 But

a+b> 2Vab and 6/3 — 10> 0, so it suffices to show that

ab? + (6V3 - 10) (2Vab + ab) — 2ab > 0

The substitution t = Vab > 0 reduces this inequality to tt 4 (sv3 — 12) # +2 (6v3-10)t> 0, “ f+ (6v3 - 12) t+2(6V3—10) > 0 The derivative of the function f(t) =0 + (6V8—12)++ 2 (6V8— 10), + >0 f#@=3 ( — (v3 — )) and has only one positive zero It is \/3 — 1 and it’s easy to see that this is a minimum is point for f in the interval [0, co) Consequently f(t) > f(V3-1) =0,

and we are done

A final observation: in fact we have

Trang 12

which shows that ƒ (£) > 0 for £ > 0

71 | Marian Tetiva | Prove that for any positive real numbers a, b,c, a®—b bì— c ce —a*| _ (a—b)* + (b—0)* + (c— a)" — 4 * a+b b+c c+a Moldova TST, 2004 First solution: First of all, we observe that right hand side can be transformed into a3 — bŸ 3 x8 1 1 3 «3 1 1 2, a+b — (a -)(5-—) +0 -' (TT) 7 (a —b)(e— b)\(a—0) (> ab) (œ + b)(ø + c)(b + c)

and so we have to prove the inequality

|l(a — ð)(b — e)(ce— a)|(ab+ be+ ca) _ 1 2

(a+b)(b + e)(e+ 4) <5 (Ye _ 80)

(œ+b+ e)(ab + be + ca) and

It is also easy to prove that (ø + ð)(b + c)(e+ a)

so we are left with IV <O1 Go oa (X2(4— 9?) > [« -%| Using the AM-GM Inequality, we reduce this inequality to the following one (3) >|H«=9| This one is easy Just observe that we can assume that a > 6 > ¢ and in this case it becomes 3 (œ — b)(a — e)(b — c) < 2 and it follows from the AM-GM Inequality (a+b+ ø)Ÿ Second solution (by Marian Tetiva):

It is easy to see that the inequality is not only cyclic, but symmetric That is why we may assume that a > b > c > 0 The idea is to use the inequality

2 2

y winyty x

4+ 5 > ——— —— >†/+†+—

eros x+y <9 5

Trang 13

That is why we can write a? — b? a’? + ab + b2 b + be -L c2 gŠ -L ae - c2 » a+b = (ape teeth a+b gether Và j2 Taote ` (a-b) (0+ $) +00) (c+ 5} — &=9 (ø+ 3) = 3692 4 In the same manner we can prove that ng < 5 ”(ø — b)? œ+b — 4 IV

and the conclusion follows

72 | Titu Andreescu | Let a,b,c be positive real numbers Prove that

(a° — a? +3)(0? —b7 4+3)(2 —c +3) > (at+b+e)®

USAMO, 2004

Solution:

We start with the inequality a° —a2+3>a?+2 6 (a?—1)(a?—1) > 0 Thus,

it remains to show that 3 II) > (2+) Using the AM-GM Inequality, one has a3 1 1 3a + + > 3 3 3 = ajt+2 bb) 4+2 42 #/[(a? + 2) We write two similar inequalities and then add up all these relations We will find that 3 [Ie +2> (Sa),

which is what we wanted

73 | Gabriel Dospinescu | Let n > 2 and #1,#s, ,#„ > 0 such that

DÓI ao

2 - = 2 4+——

(Sot): (a) owe mCn

Trang 14

Solution:

In this problem, a combination between identities and the Cauchy-Schwarz Inequality is the way to proceed So, let us start with the expression Li 4; ° (= 4th 2) | 1<i<j<n t wi We can immediately see that > (r3) *j Li 1<i<g<n l| Z mm SI + SIS | |S | Hs S| + œ —_— l| 1<i<jcn

- (E82) Es) E3)~

Thus we could find from the inequality 2 ` (+3 -›) >0 — # 1<i<j<n (9) a)ae~ Unfortunately, this is not enough So, let us try to minimize 2 Ly %¿ > (r?) x 1<i<j<n J that This could be done using the Cauchy-Schwarz Inequality: (eens Lj %; n 2 3%; x; 2 c2, (Ee) Xj — Li 2; Because » (= a 2) = 1, we deduce that ‡<?<j<n j vi

Sa) (Ea) eras

which is what we wanted to prove Of course, we should prove that we cannot have equality But equality would imply that x, = v2 = - = &,, which contradicts the

É⁄)82)-e

Trang 15

74 | Gabriel Dospinescu, Mircea Lascu, Marian Tetiva ] Prove that for any po- sitive real numbers a, Ö, c,

a2 + b2 + e? + 2abe+ 3 > (1+ ø)(1+b)(1+ e)

First solution:

Let ƒ(ø,b,e) = a? +? + c2 + abe+ 2— ø—b— e— ab— be — ca We have to prove

that all values of f are nonnegative If a,b,c > 3, then we clearly have - + : + : < 1, which means that ƒ(ø,b,e) > a? + b + cẰ+2—a—b—c >0 So, we may assume

b

that a < 3 and let m = — Easy computations show that f(a, b,c) — f(a,m,m) =

(3 — a)(b—c)* 1 > 0 and so it remains to prove that f(a,m,m) > 0, which is the same as (a+1)m? —-2(a4+1)m4+a?—a4+2>0 This is clearly true, because the discriminant of the quadratic equation is —4(@ + 1)(a — 1)2 <0 Second solution: Recall Turkevici’s Inequality z* +? + z2 +! + 2xuzt > z2? + 2z? + z2) + 12x? + z2 + 2£

for all positive real numbers x, y,z,t Taking t = 1,ø = #2,b = 9Ÿ,z = z and using the fact that 2/Vabc < abc + 1, we find the desired inequality

75 | Titu Andreescu, Zuming Feng | Let a,b,c be positive real numbers Prove

that

(2a+b+c)— (2b+a+c) (2c+a+b)? <8

2a2 + (b+c)? 26?+(at+c)? 2c3+(a+b)3 — —

USAMO, 2003 First solution:

Because the inequality is homogeneous, we can assume that a+ b+c= 3 Then

(2a+b+c)? _ a2 + 6a+9 _ M42 4a+3 ) <

Trang 16

Second solution: b + b Denote x = re y= 2 eS We have to prove that a c (x + 2)? 2z +1 5 (2-1)? 1 WT <g6 <=e©ŠS `" ` ` an ——>- But, from the Cauchy-Schwarz Inequality, we have ye ` (z++z—3)? z?+2 —z?+?2+z?+6 It remains to prove that Q(x? + y? + 2? 4 Qay + 2z + 2z+ — 6 — 6y — 6z+9) >z?+?+z”+6© ©z?+ˆ°+z”+4(zụ + 0z + zz) — 12(œ + + z) + 123 0

Now # + z + zz > 3/+22z2 > 12 (because #z > 8), so we still have to prove that

(xtytz)?+24—12(2+y+z) +12 > 0, which is equivalent to (x +y+z-—6)? > 0,

clearly true

76 Prove that for any positive real numbers x,y and any positive integers m,n,

(n—1)(m—1) (a? ty™*") + (mtn—-l)(ary"+ary™) > mnlartr—ly tytn" 7), Austrian-Polish Competition, 1995

Solution:

We transform the inequality as follows:

mx —y)(a™ Fr" — y™*P—1) > (mtn —W(a™—y™)(a" —y") & .m+n—] _ ụ n1 gm — m coy nr n ứn +? — 1) — U) — m(œ— W)_ n(z — 0) (we have assumed that x > y) The last relation can also be written # x x ey) | min tars | ear f t” "dt ¥ ¥ Ụ

and this follows from Chebyshev’s Inequality for integrals

Trang 17

with x,y, 2,t,u > 0 It is clear that 1 + 1 atabe — y tt ~ 1 tsi 1+ ab + abcd Ta T+ + Z U tạ 1 1 1 1 Writing the other relations as well, and denoting — = a,, — = aa, T— = đa, TT a4,-= x y Z u as, we have to prove that if a; > 0, then 1 ` _ 0a 1d „1 a, +agt+a5 - 3 Using the Cauchy-Schwarz Inequality, we minor the left-hand side with 4S? 2S% — (ag + a4)? — (a, + a4)? — (a3 + a5)? — (ag + a5)? — (a, + a3)?’ 5 where S = So ai By applying the Cauchy-Schwarz Inequality again for the

denominator Of the fraction, we obtain the conclusion

78 | Titu Andreescu | Prove that for any a,b,c, € (0, 3) the following inequality

holds

sỉn ø - sin(œ — Ö) - sin(œ —e) sinb-sin(6 — e) - sin(b — ø)_ sinc- sin(e — a) - sin(e — b) >0 sin(b + c) sin(c + a) sin(a + b) —

TST 2003, USA

Solution:

Let x = sina, y = sinb, z = sinc Then we have x,y,z > 0 It is easy to see that the following relations are true:

sina - sin(a — b)- sin(a — c) - sin(a + 6) « sin(a +c) = x(a? — y?) (a? — z”)

Using similar relations for the other terms, we have to prove that:

> 2(2? — y°)(a? —y*) > 0

With the substitution 2 = /u, y = Vv, z = Vw the inequality becomes » Vu(u — v)(u — w) > 0 But this follows from Schur’s Inequality

79 Prove that if a,b,c are positive real numbers then,

Va4 + b1 + c+ + Veh? + Pet 2a? > Va3b+ Bet Gat Vab? + be? + ca

KMO Summer Program Test, 2001

Solution:

It is clear that it suffices to prove the following inequalities

Trang 18

and

(Soot) (Shas?) = (Sad) (2á)

The first one follows from Schur’s Inequality

So at+abeS a > S a?b+ áp

Sab? > abe S~ a

The second one is a simple consequence of the Cauchy-Schwarz Inequality:

and the fact that

(ab + Betcha) < (ab? + b?c? +? a7)(at + bt 4+ &*) (abŸ + beŠ + ca®)? < (a2bŠ + b°e? + e?a?)(a* + b + €)

80 | Gabriel Dospinescu, Mircea Lascu | Eor a given ø% > 2 find the smallest

constant k, with the property: if a1, ,@, > 0 have product 1, then a1 Q2 4 4203 4 4 Andy, <k (a? +a2)(az +a) (a3 +.a3)(a? + a2) (a2 + øi)(aˆ + an) ” Solution: 1 Let us take first aj = đứa = - =ữạ_ 1 —=#,0ạ = —- We infer that en bk > Qe2r-1 + m„ — 2 ` 7+ — 2 "= amETTTj@TT+T) ”(+z)” ”1+ø for all z > 0 Clearly, this implies k, > ?— 2 Let us prove that ø — 2 1s a good

constant and the problem will be solved

First, we will prove that (2? + y)(y? + ©) > zw(1 + z)(1 + g) Indeed, this is the same as (x + y)(x — y)” > 0 So, it suffices to prove that

1 1 1

+ fess + —— < n-2

(1+a1)(L+az) (1+ a2)(1 +43) (1+ an)(1+ ai) ~

Trang 19

So, we have to prove the inequality n S- Chee >1, ha Mek + €p41) (p41 + Le+2) Using the Cauchy-Schwarz Inequality, we infer that n 2 de n 2 +1 k+1 > 2„ (tp + #g+1)(#g+q + #eg+2) — xẽ k=1 (tp + Le41)(Le+1 + Le42) k=1 and so it suffices to prove that n 2 Th nr Tr » «| > Sox; +23 7i + 0a k=1 k=1 k=1 k=1 But this one is equivalent to n n 2 › Liv; > 2 › %g#k+1 + ) 3g'k 3 1<i<j<n

and it is clear Thus, k, = n — 2

81 [ Vasile Cirtoaje | For any real numbers a, b,c, x,y, z prove that the inequality holds az + bụ + cz + (a3 + b2 + c2)(z2 + 2+ z3) > s(œ+b+e)(z++z) wl bo Kvant, 1989 Solution:

far ty? +22 v Hung

Let us denote # = \/ —————: Using the substitutions « = tp, y = tq and a3 -+ b2 + c2 Š Poy = tq z = tr, which imply C4+PtP=apPigtr The given inequality becomes os 2 aptbgter+a° +b +e > 5 at+b+c)(p+q+r), (at+p)?+(b+q)?4+(et+r) > =(atb+e\(ptqtr) Wl A Since 4(a+b+e)(p+qg+r)<[(a+b+e)+(p+aqg+r)}È

it suffices to prove that

(a+ p)? + (b+q)? +(ce+r) > =[(at+p) + (b+ q) + (ctr)

Wl]

—¬

Trang 20

82 [ Vasile Cirtoaje | Prove that the sides a, b, c of a triangle satisfy the inequality

3(0 4242-1) 50(2 4622) b ¢ oa a b oe

First solution:

at c After

We may assume that c is the smallest among a, b,c Then let x = b— some computations, the inequality becomes

(3a—2e)z°+ (ø +e- 1) (a—c)? > 0 © (3ø—2e)(2b—a—e)?+(4b+2e—3a)(a—e)Ÿ > 0 which follows immediately from 3ø > 2c, 4b + 2e — 3ø = 3(b+e— ø)+b—ec>0

Second solution:

Make the classical substitution a =y+2,b=z++2,c=2x+y and clear denomi- nators The problem reduces to proving that

#2 + 9 + z3 + 2(z?u + 0 )z + z +) > 3(z2 + uz + zz”)

We can of course assume that x is the smallest among x,y,z Then we can write y=xt+m,z=2+Nn with nonnegatives m and n A short computation shows that

the inequality reduces to 2x(m? — mn +n?) + m3 +n? + 2m?n — 3n?m > 0 All we need to prove is that m? +n? + 2m?n > 3n?m | (n—m)> —(n—m)m? +m? > 0 and this follows immediately from the inequality t? + 1 > 3, true for t > —1

Trang 21

But this one is not very hard, because it follows immediately by multiplying the 1 1 142) > 1+ n—-1 —— II ( #7 JIL, obtained from Huygens Inequality inequalities n—-1 Second solution: We will prove even more, that - 1 "?—1\” xy 1 l(-3)>(% 'lrz x ¿=1 , It is clear that this inequality is stronger than the initial one First, let us prove that

This follows from Jensen’s Inequality for the convex function f(x) = In(1+ 2) — In(1 — x) So, it suffices to prove that 1) - n”—1\" —>——:||a-z”>|——] - ñ lƯ PT) But a quick look shows that this is exactly the inequality proved in the solution of the problem 121

84 | Vasile Cirtoaje, Gheorghe Eckstein | Consider positive real numbers #1,Z2, ,„ such that #+zs #„ = l Prove that 1 1 1 + + +®————<Il nm—-lt+2a n—-14+ 22 m=— Ì + #„ TST 1999, Romania First solution:

Suppose the inequality is false for a certain system of n numbers Then we can find a number & > 1 and n numbers which add up to 1, let them be a;, such that

1

——- = ka; Then we have n—-1+2;

Trang 22

1

We have used here the fact that a; < n1 Now, we write | — (n — l)ay = by and

nN —

we find that ` b„ = 1 and also I[a — by) < (n — 1)”bị b„ But this contradicts

the fact that for each 7 we have 1—6; = 6) +-+-+bj-1 + bj41 +-:- +b, > (n-1) "YX by .b3-10;41 On Second solution: Let us write the inequality in the form #1 33 Ln - + +————> nm-lt+a, n-1+%2%2 nm—-14+2p This inequality follows by summing the following inequalities 1—+ 1—+ — — 1—+ 1—+ 1—+)``` — — 1—+ — 1

nh T1 cự hượ tự, dan ” n— Tần ph cự ta, 4+n The first from these inequalities is equivalent to Lt 1—+ 1—+ — + a, "+a, "+ +4n "> (n-1)x," and follows from the AM-GM Inequality Remark

Replacing the numbers 21, %2, ,%, with i Ty respectively, the in-

equality becomes as follows ma "

1 1 1

1+(m-— 1)zi T 1+(n—1)za T::*TT@n-1zaÊ \

85 [ Titu Andreescu |] Prove that for any nonnegative real numbers a, b,c such

that a? + 6? + c? + abc = 4 we have 0 < ab+ be + ca — abc < 2

USAMO, 2001

First solution (by Richard Stong):

The lower bound is not difficult Indeed, we have ab + bc + ca > 3Va2b2c2 and thus it is enough to prove that abc < 3Va2b2c2, which follows from the fact that abc < 4 The upper bound instead is hard Let us first observe that there are two numbers among the three ones, which are both greater or equal than 1 or smaller than or equal to 1 Let them be b and c Then we have

4 > 2be +a” + abe > (2—a)(2 +a) > bc(2 +a) > be < 2-4

Thus, ab+ bc+ca-—abe < ab+2—a+ac—abc and it is enough to prove the inequality

qb + 2— a+ ac— be < 2© b+c— be< 1© (b— 1)(c— 1) >0, which is true due to

Trang 23

Second solution:

We won’t prove again the lower part, since this is an easy problem Let us con- centrate on the upper bound Let a > 6b > cand leta=ax+y,b=a2-—y The

hypothesis becomes x?(2 + c) + y?(2 —c) = 4—c? and we have to prove that (2? — y?)(1—c) < 2(1 — 2c) Since y? = 2+ e— — the problem asks to

4 2 4— 2 4

prove the inequality dot 4) —c) < 2(1 — cx) Of course, we have c < 1 e

and 0 < y? = 24+c- 37? z? 2-c > x < V2-c (we have used

the fact that a > 6 > y > O and b 0 > x« > y > 0) Now, consider the

4z2 — (4— c?)

function ƒ : [0,v2— e| > R, f(x) = 201 - cx) - — ze —c) We have

l-e

f'(x) = -2c— 82- 5 < 0 and thus f is decreasing and f(x) > f(W2—c) So, we

have to prove that ƒ(v2—c) > 06 2(1-evV2 — c) > (2-0)(1-c) 6 3 > c4.2V2 —6 6 (1-V2—)? > 0 clearly true Thus, the problem is solved 86 | Titu Andreescu | Prove that for any positive real numbers a, b, c the following inequality holds at+tb+e 3 ~ Vabe < maz{(VJa — vb)", (Vb — Ve)”, (We — Va)’} TST 2000, USA Solution: A natural idea would be to assume the contrary, which means that a+0+€_ #1 <œ+b- 2Vab 3 b “TT —— Ÿabe<b+e~ 9Vữc a+b+€c 3 — Vabe < e+ø— 2V(a

Adding these inequalities, we find that

a+b+e—3WVabe > 2(a+b+c¢—Vab— Vbe— Vea)

Now, we will prove that ø+b+ e— 3Ä⁄abe < 2(a+b+e— Wab — Vbe — ca) and the

problem will be solved Since the above inequality is homogeneous, we may assume

that abc = 1 Then, it becomes 2Vab + 2V/bc + 2\/ca — a — b—c < 3 Now, using

Trang 24

87 | Kiran Kedlaya | Let a,b,c be positive real numbers Prove that

a+ Vab+ Vabe _ af a+b a+b+e

3 — 2 3 ‘

Solution (by Anh Cuong):

We have that

a+Vab+ Vabe < a+ §/ar° + Yabe Now we will prove that | b / b b at lao + Vabe < Va os By the AM-GM Inequality, we have Le 2a + 3a 3/1 2a 8a < a+b a+b+ce a+b a+b+ec_ 3 , 3b 2 — 31.1 _ 3U « Ta b+c atb+te7 3 , Le 2b + 3¢ sy, 26 3c a+b_ a+b+ec a+b a+b+ec— 3 l Now, just add them up and we have the desired inequality The equality occurs when a=b=c 88 Find the greatest constant & such that for any positive integer n which is not a square, |(1 + /n)sin(r/n)| > k Vietnamese IMO Training Camp, 1995 Solution:

We will prove that is the best constant We must clearly have k <

Trang 25

1 i) The first case is when {,/n} < 3 Of course, 1 {vn} 2 Vin Vn— T= —" 3 x and because sin x > x — S we find that sin(z{Wn}) > sin ———— > (=) — 1 (——): —— Wn—l+vn — \vn— L1+ vn 6 \VøT— 1+ vn 7T Let us prove that the last quantity 1s at least ———— This comes down to P 4 y 2(1 + Jn) 2+ vn=vn=T x2 1+ vn 3(Vn+wn=T)”” or 6(⁄n + Vn — 1)? + 3(VWn + vn — 1) > a?(1 + Vn) and ït is clear 1 1 ii) The second case is when {./n} > 3 Let er = 1— {Wn} < 5 and let n = 1 k? + p,1 < p < 2k Because {,/n} > 5 = p > k+l Then ït 1s easy to see that 1 x > —————— _ k+l+vk?+2k and so it suffices to prove that TT TT sin > k+14+Vk?+2k ~ 2114 Vk? +k) 3 Using again the inequality sing > x — 6 we infer that, 2Vk2+k— Wk2+2k— k+ 1 5 > 1” 5: l+Vk° +k 3(1+k+ Vk? + 2k) But from the Cauchy-Schwarz Inequality we have 2Vk? + k— Vk? +2k—k>0 2 Because the inequality 1 +k+vk?+ 2E)” > > (1 + Vk? + i) holds, this case is also solved 89 [ Dung Tran Nam ] Let 2, y,z > 0 such that (27 + y+ z)° = 32zryz Find the ety? +24 minimum and maximum of 7: (ct+y+z) Vietnam, 2004

First solution (by Tran Nam Dung):

Trang 26

at +y* +24 44 ¬ = (a? ty? +2) 2Š” z?y2 — (16— 2 So ay)? — 23 #)Ÿ + 4zz(œ + + z) = = 2a?— 64a + 288, extremal values of Now, we have 2

where a = xy+yz+za Because y+z = 4—2 and yz = —, we must have (4— #) x 2> which implies that 3— /5 < « < 2 Due to symmetry, we have 2, y,z € [3 — V5, This means that (a — 2)(y — 2)(z — 2) < 0 and also (— 3+ V5)( - 3+ V5)(z — 3+ v5) > 0 ? 93 |%œ Clearing paranthesis, we deduce that ae „#21 z2 +t+z? — (a— 16)?— 112 44 7 128 —1 9 1 1+ v5 383 — 165V5 (3- vs + vỗ 4) But because , we find that the extremal values of the expression are 256 » =, attained for the triples 128 yD respectively (2,1, 1) Second solution: As in the above solution, we must find the extremal values of x? + y? + z? when 1

#-+ + z = l,zz = 39° because after that the extremal values of the expression xe’ + y* + z* can be immediately found Let us make the substitution x = T1 =

b 2 b2 4 2

T2” 7 where abe = 1,a +64 2c =4 Then 2? + y?4 27 = ———— and so we

2

must find the extremal values of a? + b? + 4c” Now, a? +b? + 4c? = (4— 2e)3— ~ +4e? c

and the problem reduces to finding the maximum and minimum of 4c? — 8c — 1 where there are positive numbers a,b,c such that abc = 1,a +64 2c = 4 Of course, this 3— võ 2 , 1} But this reduces to the study vỗ a 1 — of the function f(z) = 4x? — 8z — — defined for x 4 4 comes down to (4 — 2c)? > -, or toc € | c

, which is an easy task

90 | George Tsintifas ] Prove that for any a,b,c,d > 0,

(a + b)?(b + e)?(e+ đ)?(d+ a)) > 16a?b?c2d?(œ + b+ e+ đì!

Trang 27

Solution:

Let us apply Mac-Laurin Inequality for

x = abc, y = bed, z = cda,t = dab We will find that

S| abe ° ` 3 abc - bed - của _ a2b2c2đ2 ` a 7

4 — 4 7

Thus, it is enough to prove the stronger inequality

(a+ b)(b+c)(e+d)(d+a) > (a+b6+c+4d)(abe + bed + cda + dab)

Now, let us observe that

(a + b)(b + e)(e + đ)(đ + a) = (ae + bđd + ad + be) (ac + bd + ab+ cd) = = (ac + bđ)? + 3 a?(be + bd + cả) > 4abcd + È) a®(bc + bd + cd) = = (a+b+c+d)(abe + bed + cda + dab)

And so the problem is solved

91 | Titu Andreescu, Gabriel Dospinescu |] Find the maximum value of the ex-

pression

(ab)” , (bc)" | (ca)”

l—ab 1-—be 1 -ca

where a,b,c are nonnegative real numbers which add up to 1 and n is some positive integer Solution: First, we will treat the case n > 1 We will prove that the maximum value is 1 3 gna" It is clear that ab, bc, ca < 1 and so (ab)” + (bc)” (ca)” _ 4 1—ab 1—be l—eca < 3 ((ab)* + (be)* + (ca)*) 1

Thus, we have to prove that (ab)” + (bc)” + (ca)* < 7 Let a the maximum among

a,b,c Then we have 1 m >a"(1—a)" =a"(b+c)" > a"b" + a"e" +naPb"—1e > a"b" + phe" + cha", 1 So, we have proved that in this case the maximum is at most 3.an=1' But for 1

a=b= 5 c = 0 this value is attained and this shows that the maximum value is for n > 1 Now, suppose that n = 1 In this case we have

ab 1

`

3.4n=1

Trang 28

Using the fact that a+b+c= 1, it is not difficult to prove that 1 1 1 3-2 » ab + abe 1—ab ẹ T—be* 1-ca | 1— Sab + abe - abc? b 1 ˆ ob < a With the above observations, this reduces to ` ab < 3 — 27(abc)? + 19abc 11 But using Schur’s Inequality we infer that 1 3 ab < oo

We will prove that »

and so it is enough to show that

9abc + 1 < 3 — 27(abe)? + 19abec

4 — 11

& 108(abc)* + 23abe < 1,

1

which is true because abc < a7

Hence for n = 1, the maximum value is 3 attained for a = b= ce 92 Let a,b,c be positive real numbers Prove that 1 + 1 + 1 > 3 a(l+b6) b+e) c(1+a) — Ÿabe(1+ Vabe) Solution: The following observation is crucial 1 1 1 l+abe+a+ab (+ abe) (T+ ota) +3 = 2 annp _ l+a b(c + 1) 7 aa td I+b ` We use now twice the AM-GM Inequality to find that ys +® e1) > 3— + 3Ÿabe a (1+) 1+b — VYabe And so we are left with the inequality 3 + 3Vabec — 3 3 Vabc 3 > 3

1+ abe ~ Vabc(1 + Vabc)

which is in fact an identity!

93 [ Dung Tran Nam ] Prove that for any real numbers a, b,c such that a? +6? +

cẰ =9,

2(z+b+c)— abc < 10

Trang 29

First solution (by Gheorghe Eckstein):

Because max{a, b,c} < 3 and |abc| < 10, it is enough to consider only the cases

when a,b,c > 0 or exactly of the three numbers is negative First, we will suppose that a,b,c are nonnegative If abc > 1, then we are done, because

2(a+b+c) — abe < 24/3(a2 + b2 + c2) — 1 < 10

Otherwise, we may assume that a < 1 In this case we have

b2 -+ œ2

St +) =dác <9 [s3 = = 2a + 2V18 — 2a? < 10

Now, assume that not all three numbers are nonnegative and let c < 0

Thus, the problem reduces to proving that for any nonnegative x,y,z whose sum of squares is 9 we have 4(a + y — z) + 2ayz < 20 But we can write this as

(z—2)”+(/—2)”+(z—1)Ÿ > 2zwz — 6z — 2 Because 2zz—6z—2 < z(y?+z”?)—6z—2 =

—Z + 3z — 2 = -(z — 1)?(z + 2) < 0, the inequality follows

Second solution:

Of course, we have |a|, |b|,|c| < 3 and |a + b + c|,|abe| < 33 Also, we may

assume of course that a,b,c are non-zero and that a < ö < e le < 0 then we have

2(a+b+e)—abe < —abe < 3V3 < 10 Also, if a < 6 < 0 < cthen we have 2(a+b+c) <

2c <6 < 10+ abc because abc > 0 If a << 0 < b < c, using the Cauchy-Schwarz

Inequality we find that 26+ 2c—a < 9 Thus, 2(a+6+c) = 2b+2c—a+3a <9+3a

and it remains to prove that, 3a — 1 < abe But a < 0 and 2be < 9 — a”, so that it

— > 3a— 1© (a + 1) (a — 2) < 0, which follows So, we

Just have to threat the case 0 < a < b < c In this case we have 2b + 2c + a < 9 and

2(z+b+c) <9+a So, we need to prove that a < 1 + abe This is clear iÍ a << 1 and

if a > 1 we have b,c > 1 and the inequality is again Thus, the problem is solved

remains to show that

Trang 30

we get

0z + ø + z + zz >3 4,

> 0 we distinguish

(c= 1)*

with equality if and only if abe = 1 Because y+z = tL bt

two cases a) x> 0, yz < 0; b)x> 0,y> 0,z> 0 ,

a) x> 0, yz < 0 We have xyz < 0 and Írom #2 + ø + z + zz > 4 we get z Ặ+z+zz> 4> 3 b) x> 0,y> 0,z> 0 We denote z + yz + zx = 3d? with d > 0 From the mean inequalities we have z + z + zz > 33/+z292z2, from which we deduce that xyz < d? On the basis of this result, from the inequality 2z + #U + 0z + zz > 4, we obtain đ + 3d? > 4, (d— 1)(d+ 2)? > 0,d > 10 z + z + zz > 3 With this the given inequality is proved We have equality in the caseea=b=c=l Second solution: Letu=a2+lv=y+1,w=2z+4+1 Then we have 1 uw =ututw+tabe+ Tà >utvtw+2 abe

Now, consider the function f(t) = 2¢°+t?(u+v+w)—uvw Because Jima f(t) = œ and

f() < 0, we can find a real number r > 1 such that f(r) = 0 Consider the numbers

m=-,n= 0 —,p= — They verify mnp = m+n+p+2 we deduce from problem 49

that Am +np-+pm È > 2(m-+n.+p) > ưu + 0u + tu > 2r{(u + +) > 2(u +0)

But because u = x+1,v = y+1,w = z+1, this last relation is equivalent to sy +yz+ zx > 3, which is what we wanted

95 [ Gabriel Dospinescu |] Let n be an integer greater than 2 Find the greatest

Ngày đăng: 28/10/2013, 20:15

TỪ KHÓA LIÊN QUAN

w